Quantcast
  • Register
PhysicsOverflow is a next-generation academic platform for physicists and astronomers, including a community peer review system and a postgraduate-level discussion forum analogous to MathOverflow.

Welcome to PhysicsOverflow! PhysicsOverflow is an open platform for community peer review and graduate-level Physics discussion.

Please help promote PhysicsOverflow ads elsewhere if you like it.

News

PO is now at the Physics Department of Bielefeld University!

New printer friendly PO pages!

Migration to Bielefeld University was successful!

Please vote for this year's PhysicsOverflow ads!

Please do help out in categorising submissions. Submit a paper to PhysicsOverflow!

... see more

Tools for paper authors

Submit paper
Claim Paper Authorship

Tools for SE users

Search User
Reclaim SE Account
Request Account Merger
Nativise imported posts
Claim post (deleted users)
Import SE post

Users whose questions have been imported from Physics Stack Exchange, Theoretical Physics Stack Exchange, or any other Stack Exchange site are kindly requested to reclaim their account and not to register as a new user.

Public \(\beta\) tools

Report a bug with a feature
Request a new functionality
404 page design
Send feedback

Attributions

(propose a free ad)

Site Statistics

205 submissions , 163 unreviewed
5,047 questions , 2,200 unanswered
5,345 answers , 22,709 comments
1,470 users with positive rep
816 active unimported users
More ...

  Dirac, Weyl and Majorana Spinors

+ 6 like - 0 dislike
3627 views

To get to the point - what's the defining differences between them? Alas, my current understanding of a spinor is limited. All I know is that they are used to describe fermions (?), but I'm not sure why?

Although I should probably grasp the above first, what is the difference between Dirac, Weyl and Majorana spinors? I know that there are similarities (as in overlaps) and that the Dirac spinor is a solution to the Dirac equation etc. But what's their mathematical differences, their purpose and their importance?

(It might be good to note that I'm coming from a string theory perspective. Plus I've exhausted Wikipedia here.)

This post imported from StackExchange Physics at 2015-03-04 16:08 (UTC), posted by SE-user Phibert
asked Mar 11, 2014 in Theoretical Physics by Phibert (30 points) [ no revision ]

2 Answers

+ 6 like - 0 dislike

Recall a Dirac spinor which obeys the Dirac Lagrangian

$$\mathcal{L} = \bar{\psi}(i\gamma^{\mu}\partial_\mu -m)\psi.$$

The Dirac spinor is a four-component spinor, but may be decomposed into a pair of two-component spinors, i.e. we propose

$$\psi = \left( \begin{array}{c} u_+\\ u_-\end{array}\right),$$

and the Dirac Lagrangian becomes,

$$\mathcal{L} = iu_{-}^{\dagger}\sigma^{\mu}\partial_{\mu}u_{-} + iu_{+}^{\dagger}\bar{\sigma}^{\mu}\partial_{\mu}u_{+} -m(u^{\dagger}_{+}u_{-} + u_{-}^{\dagger}u_{+})$$

where $\sigma^{\mu} = (\mathbb{1},\sigma^{i})$ and $\bar{\sigma}^{\mu} = (\mathbb{1},-\sigma^{i})$ where $\sigma^{i}$ are the Pauli matrices and $i=1,..,3.$ The two-component spinors $u_{+}$ and $u_{-}$ are called Weyl or chiral spinors. In the limit $m\to 0$, a fermion can be described by a single Weyl spinor, satisfying e.g.

$$i\bar{\sigma}^{\mu}\partial_{\mu}u_{+}=0.$$

Majorana fermions are similar to Weyl fermions; they also have two-components. But they must satisfy a reality condition and they must be invariant under charge conjugation. When you expand a Majorana fermion, the Fourier coefficients (or operators upon canonical quantization) are real. In other words, a Majorana fermion $\psi_{M}$ may be written in terms of Weyl spinors as,

$$\psi_M = \left( \begin{array}{c} u_+\\ -i \sigma^2u^\ast_+\end{array}\right).$$

Majorana spinors are used frequently in supersymmetric theories. In the Wess-Zumino model - the simplest SUSY model - a supermultiplet is constructed from a complex scalar, auxiliary pseudo-scalar field, and Majorana spinor precisely because it has two degrees of freedom unlike a Dirac spinor. The action of the theory is simply,

$$S \sim - \int d^4x \left( \frac{1}{2}\partial^\mu \phi^{\ast}\partial_\mu \phi + i \psi^{\dagger}\bar{\sigma}^\mu \partial_\mu \psi + |F|^2 \right)$$

where $F$ is the auxiliary field, whose equations of motion set $F=0$ but is necessary on grounds of consistency due to the degrees of freedom off-shell and on-shell.

This post imported from StackExchange Physics at 2015-03-04 16:08 (UTC), posted by SE-user JamalS
answered Mar 11, 2014 by JamalS (895 points) [ no revision ]
So am I right in saying all Majorana spinors are Weyl spinors, but not the other way around? And Weyl spinors, and therefore Majorana spinors as well, are subsets of Dirac spinors?

This post imported from StackExchange Physics at 2015-03-04 16:08 (UTC), posted by SE-user Phibert
I think you got it the other way around. All Majorana spinors are constructed from Weyl spinors, but Weyl spinors are not Majorana spinors.

This post imported from StackExchange Physics at 2015-03-04 16:08 (UTC), posted by SE-user user32361
This is what I said?

This post imported from StackExchange Physics at 2015-03-04 16:08 (UTC), posted by SE-user Phibert
+ 1 like - 0 dislike

After you will learn more about spinors, you will see that all spinors belong to the $\left(\frac{1}{2}, 0\right) + \left( 0, \frac{1}{2}\right)$ representation of the $SL(2,C)$ group, which is the double cover of the lorentz group $SO(3,1)$. The idea is to find representations of a simply connected covering group which in this case is $SL(2,C)$, the local structure given by the lie algebraic commutation relation remains the same.

Spinorial equations allow to extract Lorentz-invariant subspaces in the overall space of $\left(\frac{1}{2}, 0\right) + \left( 0, \frac{1}{2}\right)$ representation.

Both Dirac and Majorana spinors belong to $\left(\frac{1}{2}, 0\right) + \left( 0, \frac{1}{2}\right)$ representation of $SL(2,C)$ group, but they are only subspaces of it. For instance, Majorana spinors are all electrically neutral (i.e. remain invariant under charge conjugation). Similarly, Dirac spinors are "magnetically neutral".

Weil spinors belong to either $\left(\frac{1}{2}, 0\right)$ or $\left( 0, \frac{1}{2}\right)$ subspaces. Unlike Dirac and Majorana spinors, they might be considered as 2-component spinors. But this is also a limitation, because some special Lorentz transformations cannot be applied to these spinors.

This post imported from StackExchange Physics at 2015-03-04 16:08 (UTC), posted by SE-user Murod Abdukhakimov
answered Mar 11, 2014 by Murod Abdukhakimov (85 points) [ no revision ]
Your first line is another issue I've been struggling to comprehend for a while now. Care to explain what you mean by $(\frac{1}{2},0)+(0,\frac{1}{2})$ and $SL(2,C)$? Representations just seem beyond me at the moment.

This post imported from StackExchange Physics at 2015-03-04 16:08 (UTC), posted by SE-user Phibert
@Murod: Could you elaborate on what you mean by Dirac spinors are "magnetically neutral"?

This post imported from StackExchange Physics at 2015-03-04 16:08 (UTC), posted by SE-user Siva
@user13223423: The $so(3,1)$ lie algebra decomposes as $su(2)_L \oplus su(2)_R$. So any representation of $su(3,1)$ must be a tensor product of representations of the two subalgebras. Weyl spinors are in the "fundamental" rep of one of the $su(2)$ while they're in the trivial representation of the other. That is denoted by $({\frac{1}{2}}_L , 0_R)$ or the other way around. The Dirac and Majorana representations happen to be a linear combination of the two Weyl representations (with maybe other conditions).

This post imported from StackExchange Physics at 2015-03-04 16:08 (UTC), posted by SE-user Siva
There is a good explanation about representations of $\mathfrak{so}(3,1)$ in Weinberg's book. @Siva, you mean direct sum, not tensor product. The direct sum of (1/2,0) and (0,1/2) is the Dirac spinors, but their tensor product is (1/2,1/2) which is the 4-vector representation. As for the special Lorentz transformations, parity takes left-handed Weyl spinors to right-handed and the other way around.

This post imported from StackExchange Physics at 2015-03-04 16:08 (UTC), posted by SE-user Robin Ekman
Whoops! That was a bad slip on my part. Thanks for the correction @RobinEkman. It is indeed the direct sum.

This post imported from StackExchange Physics at 2015-03-04 16:08 (UTC), posted by SE-user Siva
@Murod: Shoudn't your first sentence read double cover of $SO(3,1)$, spinors are representations of $SL(2,C)$ which is the double cover of $SO(3,1)$ - the lorentz group also called projective representations of the lorentz group for the same reason.

This post imported from StackExchange Physics at 2015-03-04 16:08 (UTC), posted by SE-user user7757
@ramanujan_dirac: You are absolutely right. But I did not write that "double cover", someone edited my post.

This post imported from StackExchange Physics at 2015-03-04 16:08 (UTC), posted by SE-user Murod Abdukhakimov

Your answer

Please use answers only to (at least partly) answer questions. To comment, discuss, or ask for clarification, leave a comment instead.
To mask links under text, please type your text, highlight it, and click the "link" button. You can then enter your link URL.
Please consult the FAQ for as to how to format your post.
This is the answer box; if you want to write a comment instead, please use the 'add comment' button.
Live preview (may slow down editor)   Preview
Your name to display (optional):
Privacy: Your email address will only be used for sending these notifications.
Anti-spam verification:
If you are a human please identify the position of the character covered by the symbol $\varnothing$ in the following word:
p$\hbar$ysics$\varnothing$verflow
Then drag the red bullet below over the corresponding character of our banner. When you drop it there, the bullet changes to green (on slow internet connections after a few seconds).
Please complete the anti-spam verification




user contributions licensed under cc by-sa 3.0 with attribution required

Your rights
...